User Preferences

Content preview.

Arcu felis bibendum ut tristique et egestas quis:

  • Ut enim ad minim veniam, quis nostrud exercitation ullamco laboris
  • Duis aute irure dolor in reprehenderit in voluptate
  • Excepteur sint occaecat cupidatat non proident

Keyboard Shortcuts

25.3 - calculating sample size.

Before we learn how to calculate the sample size that is necessary to achieve a hypothesis test with a certain power, it might behoove us to understand the effect that sample size has on power. Let's investigate by returning to our IQ example.

Example 25-3 Section  

Let \(X\) denote the IQ of a randomly selected adult American. Assume, a bit unrealistically again, that \(X\) is normally distributed with unknown mean \(\mu\) and (a strangely known) standard deviation of 16. This time, instead of taking a random sample of \(n=16\) students, let's increase the sample size to \(n=64\). And, while setting the probability of committing a Type I error to \(\alpha=0.05\), test the null hypothesis \(H_0:\mu=100\) against the alternative hypothesis that \(H_A:\mu>100\).

What is the power of the hypothesis test when \(\mu=108\), \(\mu=112\), and \(\mu=116\)?

Setting \(\alpha\), the probability of committing a Type I error, to 0.05, implies that we should reject the null hypothesis when the test statistic \(Z\ge 1.645\), or equivalently, when the observed sample mean is 103.29 or greater:

\( \bar{x} = \mu + z \left(\dfrac{\sigma}{\sqrt{n}} \right) = 100 +1.645\left(\dfrac{16}{\sqrt{64}} \right) = 103.29\)

Therefore, the power function \K(\mu)\), when \(\mu>100\) is the true value, is:

\( K(\mu) = P(\bar{X} \ge 103.29 | \mu) = P \left(Z \ge \dfrac{103.29 - \mu}{16 / \sqrt{64}} \right) = 1 - \Phi \left(\dfrac{103.29 - \mu}{2} \right)\)

Therefore, the probability of rejecting the null hypothesis at the \(\alpha=0.05\) level when \(\mu=108\) is 0.9907, as calculated here:

\(K(108) = 1 - \Phi \left( \dfrac{103.29-108}{2} \right) = 1- \Phi(-2.355) = 0.9907 \)

And, the probability of rejecting the null hypothesis at the \(\alpha=0.05\) level when \(\mu=112\) is greater than 0.9999, as calculated here:

\( K(112) = 1 - \Phi \left( \dfrac{103.29-112}{2} \right) = 1- \Phi(-4.355) = 0.9999\ldots \)

And, the probability of rejecting the null hypothesis at the \(\alpha=0.05\) level when \(\mu=116\) is greater than 0.999999, as calculated here:

\( K(116) = 1 - \Phi \left( \dfrac{103.29-116}{2} \right) = 1- \Phi(-6.355) = 0.999999... \)

In summary, in the various examples throughout this lesson, we have calculated the power of testing \(H_0:\mu=100\) against \(H_A:\mu>100\) for two sample sizes ( \(n=16\) and \(n=64\)) and for three possible values of the mean ( \(\mu=108\), \(\mu=112\), and \(\mu=116\)). Here's a summary of our power calculations:

As you can see, our work suggests that for a given value of the mean \(\mu\) under the alternative hypothesis, the larger the sample size \(n\), the greater the power \(K(\mu)\) . Perhaps there is no better way to see this than graphically by plotting the two power functions simultaneously, one when \(n=16\) and the other when \(n=64\):

As this plot suggests, if we are interested in increasing our chance of rejecting the null hypothesis when the alternative hypothesis is true, we can do so by increasing our sample size \(n\). This benefit is perhaps even greatest for values of the mean that are close to the value of the mean assumed under the null hypothesis. Let's take a look at two examples that illustrate the kind of sample size calculation we can make to ensure our hypothesis test has sufficient power.

Example 25-4 Section  

corn field

Let \(X\) denote the crop yield of corn measured in the number of bushels per acre. Assume (unrealistically) that \(X\) is normally distributed with unknown mean \(\mu\) and standard deviation \(\sigma=6\). An agricultural researcher is working to increase the current average yield from 40 bushels per acre. Therefore, he is interested in testing, at the \(\alpha=0.05\) level, the null hypothesis \(H_0:\mu=40\) against the alternative hypothesis that \(H_A:\mu>40\). Find the sample size \(n\) that is necessary to achieve 0.90 power at the alternative \(\mu=45\).

As is always the case, we need to start by finding a threshold value \(c\), such that if the sample mean is larger than \(c\), we'll reject the null hypothesis:

That is, in order for our hypothesis test to be conducted at the \(\alpha=0.05\) level, the following statement must hold (using our typical \(Z\) transformation):

\(c = 40 + 1.645 \left( \dfrac{6}{\sqrt{n}} \right) \) (**)

But, that's not the only condition that \(c\) must meet, because \(c\) also needs to be defined to ensure that our power is 0.90 or, alternatively, that the probability of a Type II error is 0.10. That would happen if there was a 10% chance that our test statistic fell short of \(c\) when \(\mu=45\), as the following drawing illustrates in blue:

This illustration suggests that in order for our hypothesis test to have 0.90 power, the following statement must hold (using our usual \(Z\) transformation):

\(c = 45 - 1.28 \left( \dfrac{6}{\sqrt{n}} \right) \) (**)

Aha! We have two (asterisked (**)) equations and two unknowns! All we need to do is equate the equations, and solve for \(n\). Doing so, we get:

\(40+1.645\left(\frac{6}{\sqrt{n}}\right)=45-1.28\left(\frac{6}{\sqrt{n}}\right)\) \(\Rightarrow 5=(1.645+1.28)\left(\frac{6}{\sqrt{n}}\right), \qquad \Rightarrow 5=\frac{17.55}{\sqrt{n}}, \qquad n=(3.51)^2=12.3201\approx 13\)

Now that we know we will set \(n=13\), we can solve for our threshold value c :

\( c = 40 + 1.645 \left( \dfrac{6}{\sqrt{13}} \right)=42.737 \)

So, in summary, if the agricultural researcher collects data on \(n=13\) corn plots, and rejects his null hypothesis \(H_0:\mu=40\) if the average crop yield of the 13 plots is greater than 42.737 bushels per acre, he will have a 5% chance of committing a Type I error and a 10% chance of committing a Type II error if the population mean \(\mu\) were actually 45 bushels per acre.

Example 25-5 Section  

politician

Consider \(p\), the true proportion of voters who favor a particular political candidate. A pollster is interested in testing at the \(\alpha=0.01\) level, the null hypothesis \(H_0:9=0.5\) against the alternative hypothesis that \(H_A:p>0.5\). Find the sample size \(n\) that is necessary to achieve 0.80 power at the alternative \(p=0.55\).

In this case, because we are interested in performing a hypothesis test about a population proportion \(p\), we use the \(Z\)-statistic:

\(Z = \dfrac{\hat{p}-p_0}{\sqrt{\frac{p_0(1-p_0)}{n}}} \)

Again, we start by finding a threshold value \(c\), such that if the observed sample proportion is larger than \(c\), we'll reject the null hypothesis:

That is, in order for our hypothesis test to be conducted at the \(\alpha=0.01\) level, the following statement must hold:

\(c = 0.5 + 2.326 \sqrt{ \dfrac{(0.5)(0.5)}{n}} \) (**)

But, again, that's not the only condition that c must meet, because \(c\) also needs to be defined to ensure that our power is 0.80 or, alternatively, that the probability of a Type II error is 0.20. That would happen if there was a 20% chance that our test statistic fell short of \(c\) when \(p=0.55\), as the following drawing illustrates in blue:

This illustration suggests that in order for our hypothesis test to have 0.80 power, the following statement must hold:

\(c = 0.55 - 0.842 \sqrt{ \dfrac{(0.55)(0.45)}{n}} \) (**)

Again, we have two (asterisked (**)) equations and two unknowns! All we need to do is equate the equations, and solve for \(n\). Doing so, we get:

\(0.5+2.326\sqrt{\dfrac{0.5(0.5)}{n}}=0.55-0.842\sqrt{\dfrac{0.55(0.45)}{n}} \\ 2.326\dfrac{\sqrt{0.25}}{\sqrt{n}}+0.842\dfrac{\sqrt{0.2475}}{\sqrt{n}}=0.55-0.5 \\ \dfrac{1}{\sqrt{n}}(1.5818897)=0.05 \qquad \Rightarrow n\approx \left(\dfrac{1.5818897}{0.05}\right)^2 = 1000.95 \approx 1001 \)

Now that we know we will set \(n=1001\), we can solve for our threshold value \(c\):

\(c = 0.5 + 2.326 \sqrt{\dfrac{(0.5)(0.5)}{1001}}= 0.5367 \)

So, in summary, if the pollster collects data on \(n=1001\) voters, and rejects his null hypothesis \(H_0:p=0.5\) if the proportion of sampled voters who favor the political candidate is greater than 0.5367, he will have a 1% chance of committing a Type I error and a 20% chance of committing a Type II error if the population proportion \(p\) were actually 0.55.

Incidentally, we can always check our work! Conducting the survey and subsequent hypothesis test as described above, the probability of committing a Type I error is:

\(\alpha= P(\hat{p} >0.5367 \text { if } p = 0.50) = P(Z > 2.3257) = 0.01 \)

and the probability of committing a Type II error is:

\(\beta = P(\hat{p} <0.5367 \text { if } p = 0.55) = P(Z < -0.846) = 0.199 \)

just as the pollster had desired.

We've illustrated several sample size calculations. Now, let's summarize the information that goes into a sample size calculation. In order to determine a sample size for a given hypothesis test, you need to specify:

The desired \(\alpha\) level, that is, your willingness to commit a Type I error.

The desired power or, equivalently, the desired \(\beta\) level, that is, your willingness to commit a Type II error.

A meaningful difference from the value of the parameter that is specified in the null hypothesis.

The standard deviation of the sample statistic or, at least, an estimate of the standard deviation (the "standard error") of the sample statistic.

  • Skip to secondary menu
  • Skip to main content
  • Skip to primary sidebar

Statistics By Jim

Making statistics intuitive

Understanding Significance Levels in Statistics

By Jim Frost 30 Comments

Significance levels in statistics are a crucial component of hypothesis testing . However, unlike other values in your statistical output, the significance level is not something that statistical software calculates. Instead, you choose the significance level. Have you ever wondered why?

In this post, I’ll explain the significance level conceptually, why you choose its value, and how to choose a good value. Statisticians also refer to the significance level as alpha (α).

The Greek sympol of alpha, which represents the significance level.

Your sample data provide evidence for an effect. The significance level is a measure of how strong the sample evidence must be before determining the results are statistically significant. Because we’re talking about evidence, let’s look at a courtroom analogy.

Related posts : Hypothesis Test Overview and Difference between Descriptive and Inferential Statistics

Evidentiary Standards in the Courtroom

Criminal cases and civil cases vary greatly, but they both require a minimum amount of evidence to convince a judge or jury to prove a claim against the defendant. Prosecutors in criminal cases must prove the defendant is guilty “beyond a reasonable doubt,” whereas plaintiffs in a civil case must present a “preponderance of the evidence.” These terms are evidentiary standards that reflect the amount of evidence that civil and criminal cases require.

For civil cases, most scholars define a preponderance of evidence as meaning that at least 51% of the evidence shown supports the plaintiff’s claim. However, criminal cases are more severe and require stronger evidence, which must go beyond a reasonable doubt. Most scholars define that evidentiary standard as being 90%, 95%, or even 99% sure that the defendant is guilty.

In statistics, the significance level is the evidentiary standard. For researchers to successfully make the case that the effect exists in the population, the sample must contain a sufficient amount of evidence.

In court cases, you have evidentiary standards because you don’t want to convict innocent people.

In hypothesis tests, we have the significance level because we don’t want to claim that an effect or relationship exists when it does not exist.

Significance Levels as an Evidentiary Standard

In statistics, the significance level defines the strength of evidence in probabilistic terms. Specifically, alpha represents the probability that tests will produce statistically significant results when the null hypothesis is correct. Rejecting a true null hypothesis is a type I error . And, the significance level equals the type I error rate. You can think of this error rate as the probability of a false positive. The test results lead you to believe that an effect exists when it actually does not exist.

Obviously, when the null hypothesis is correct, we want a low probability that hypothesis tests will produce statistically significant results. For example, if alpha is 0.05, your analysis has a 5% chance of producing a significant result when the null hypothesis is correct.

Just as the evidentiary standard varies by the type of court case, you can set the significance level for a hypothesis test depending on the consequences of a false positive. By changing alpha, you increase or decrease the amount of evidence you require in the sample to conclude that the effect exists in the population.

Learn more about Statistical Significance: Definition & Meaning .

Changing Significance Levels

Because 0.05 is the standard alpha, we’ll start by adjusting away from that value. Typically, you’ll need a good reason to change the significance level to something other than 0.05. Also, note the inverse relationship between alpha and the amount of required evidence. For instance, increasing the significance level from 0.05 to 0.10 lowers the evidentiary standard. Conversely, decreasing it from 0.05 to 0.01 increases the standard. Let’s look at why you would consider changing alpha and how it affects your hypothesis test.

Increasing the Significance Level

Imagine you’re testing the strength of party balloons. You’ll use the test results to determine which brand of balloons to buy. A false positive here leads you to buy balloons that are not stronger. The drawbacks of a false positive are very low. Consequently, you could consider lessening the amount of evidence required by changing the significance level to 0.10. Because this change decreases the amount of required evidence, it makes your test more sensitive to detecting differences, but it also increases the chance of a false positive from 5% to 10%.

Decreasing the Significance Level

Conversely, imagine you’re testing the strength of fabric for hot air balloons. A false positive here is very risky because lives are on the line! You want to be very confident that the material from one manufacturer is stronger than the other. In this case, you should increase the amount of evidence required by changing alpha to 0.01. Because this change increases the amount of required evidence, it makes your test less sensitive to detecting differences, but it decreases the chance of a false positive from 5% to 1%.

It’s all about the tradeoff between sensitivity and false positives!

In conclusion, a significance level of 0.05 is the most common. However, it’s the analyst’s responsibility to determine how much evidence to require for concluding that an effect exists. How problematic is a false positive? There is no single correct answer for all circumstances. Consequently, you need to choose the significance level!

While the significance level indicates the amount of evidence that you require, the p-value represents the strength of the evidence that exists in your sample. When your p-value is less than or equal to the significance level, the strength of the sample evidence meets or exceeds your evidentiary standard for rejecting the null hypothesis and concluding that the effect exists.

While this post looks at significance levels from a conceptual standpoint, learn about the significance level and p-values using a graphical representation of how hypothesis tests work. Additionally, my post about the types of errors in hypothesis testing takes a deeper look at both Type 1 and Type II errors, and the tradeoffs between them.

Share this:

how does sample size influence hypothesis testing results

Reader Interactions

' src=

June 3, 2021 at 3:30 am

Hi Jim! Greetings of the day! I we use 95%CI along with 10% absolute precision in order to calculate sample size? For example; A two-stage cluster sampling technique was executed to select the herd (primary unit) and individual dairy cattle (secondary unit). The number of herds required for the study was determined using the cluster formula described by (Ferrari et al., 2016) with an assumption of expected prevalence of BVD and IBR 11.7% (Asmare et al., 2013) and 81.8% (Sibhat et al., 2018) respectively, and 10% absolute precision with a 95% confidence interval (CI). Is this correct? I did it actually to reduce my sample size….to minimize logistics. Hope you will answer soon! With regards

' src=

May 31, 2021 at 7:34 am

Hi Jim, It’s been hard for me to understand why we need significance levels and p-values before I read this post. Thanks for your friendly guide! All my old knowledge about significance levels was based on that significance levels are the maxima of probabilities of type I errors. With type I errors being the mostly unwanted, I usually think of significance levels as measures of how aggressive we are. The higher a significance level is, the more tolerance of a type one error exists, and the more likely we are to reject the null hypothesis by mistake, because we are aggressive enough. But this appreciation seems reasonable. What do you think? Look forward to your reply.

' src=

June 1, 2021 at 12:09 am

They are slippery concepts for sure!

The significance level = the Type I error rate.

You’ve got the right idea. I guess what I’d add is, why is a higher significance level more aggressive? It’s because we’re willing to make a decision based on weaker evidence. I like to think of the significance level as an evidentiary standard. How much sample evidence do you need to decide an effect exists in the population? If you are willing to accept weaker evidence, then of course it’ll be easier for you to conclude that an effect exists and also it’ll be more likely that you’re wrong.

That’s what you’re saying, but thinking about it in terms of the required amount of evidence to draw that conclusion helps clarify the effects of increasing or lowering the significance level. If you require stronger evidence (lower significance level), it’s harder to reject the null but when you do, you’re more likely to be correct. When you require weaker evidence (higher significance level), it’s easier to reject the null but it increases the chances you’ll be incorrect.

I hope that helps!

' src=

April 16, 2021 at 5:23 am

Hi, Jim! Thank you so much for the article. Do you have literature or references that said the reasons for choosing confidence level of 90% in quantitative research or social research? Because my lecturer told I have to include theoretical reasons from literature about my selection to choose convidence level of 90% in quantitative research. Thank you. Best regards.

' src=

April 12, 2021 at 4:32 am

Hello Sir jim!

If I gonna gonna decrease my significance level alpha from 0.10 to 0.05, do I have to re-compute my sample size? Note: this is a conducted study and my panel suddenly wants to change my alpha level to be less than 0.1. You’ve said that if I lower my alpha level, the analysis will have lower statistical power, meaning the results will be questionable??

April 13, 2021 at 1:23 am

Using a significance level of 0.10 is unusual. I’m not surprised they want to lower it!

I’m not sure at what point you’re at for your study. Are you at the planning stages and haven’t collected data yet? If so, yes, you should do a power analysis to estimate a good sample size . You’ll need to include the significance level in the power analysis. Using a lower significance level will cause your sample size to increase to maintain a constant level of statistical power.

If you already have your data and are just deciding your significance level (which you should’ve done before collecting any data) before analyzing the data, lowering the significance level will reduce the statistical power of the analysis. However, it doesn’t make the results questionable. You are balancing a set of risks. Specifically, you’re reducing the risk of a Type I error but increasing the risk of a Type II error. That’s all a normal part of statistical analysis. Read my post about The Types of Error in Hypothesis Testing to understand that tradeoff. It’s a balancing act!

Using a significance level of 0.05 is standard and almost always a good decision. Don’t worry about it messing up your results. 🙂

' src=

April 11, 2021 at 8:22 pm

Hi Jim and Others,

I find the discussions on choosing the significance level, and would like to inform you of my recent works on this issue:

1. Kim, J. H., Choi, I., 2021, Choosing the Level of Significance: A Decision-Theoretic Approach, Abacus: A Journal of Accounting, Finance and Business Studies. 57 (1), 27-71,

2. Kim, J. H, 2020, Decision-theoretic hypothesis testing: A primer with R package OptSig, The American Statistician, 74 (4), 370-379.

if you have any questions, please feel free to contact me.

April 11, 2021 at 7:31 am

Yup, this helps a lot! Thank you, Sir Jim!

April 11, 2021 at 4:17 pm

You’re very welcome! 🙂

April 9, 2021 at 10:54 pm

Does changing the alpha level in the conducted study would affect the sample size? Or can you just simply change alpha 0.1 to 0.05 like nothing? What steps should I consider with this kind of scenario? Thank you!

April 10, 2021 at 12:49 am

Hi Maverick,

You’re free to change the significance level (alpha) however you should have good reasons as I discuss in this article. There are implications for your choice. If you increase alpha, your analysis has more statistical power to detect findings but you’ll also have more false positives. On the other hand, if you lower alpha, your analysis has lower statistical power but there will be fewer false positives. Read this post to learn about all that in more detail.

As for sample size, well, there are several factors involved in determining a good sample size. But, if you increase alpha to 0.10, you could use a smaller sample size to detect a specific effect size while maintaining statistical power. However, as I mention, you do that at the risk of increasing false positives. And, generally speaking, 0.10 is considered too high and would often not be taken seriously because it represents a weak standard of evidence. Again, there are implications to such a decision.

' src=

February 12, 2021 at 11:31 am

Jim, thank you for your Q&A on Statistical p value questions. I know that significance levels are set by the statistician. My question is whether a p value of, say 0.103, when rounded to the second decimal point is 0.10 and 10% significant. Would you agree with this, i.e., would the rounding issue work in this example?

February 12, 2021 at 2:51 pm

There’s really two issues related to your question, even though you’re asking about only one of them. Let me start with the other question you’re not asking.

Is a significance level of 0.10 ok to use? It’s not the standard level of 0.05. If you were to use a significance level of 0.05, then a p-value of 0.049 would be significant. However, a p-value in that range does not really provide strong evidence that an effect exists in the population. In other words, there’s a relatively high chance of a false positive even in that p-value region. For more details about that issue, I recommend reading my post about interpreting p-values . Pay particular attention to the table near the end.

You can imagine that if a p-value around 0.049 is weak evidence, then a p-value near 0.10, plus or minus a little bit, is extremely weak! I’d only use a significance level of 0.10 if there’s mainly just an upside to detecting an effect but no downside if it’s a false positive. Be aware that while all studies that are significant at the 0.10 significance level will have a false positive rate of 10%, a study with a p-value near the cutoff value will have a higher false positive rate than that (Read my link above).

Now, on to your question. If you’re already using a significance level that high (allowing weak evidence), there’s probably little difference between a 0.103 and 0.10. You’ve already accepted the high chances of a false positive. So, in practice, there’s very little difference. However, you might well run into strong opinions about the matter. Some statisticians will say, “No way! It’s a sharp cutoff!” However, I have seen even in peer reviewed journals wording about “nearly significant” and “almost significant.” Yours would fit that. However, I’m guessing your study is not be for a peer reviewed journal because they typically don’t accept significance levels of 0.10.

So, for your specific case, if you’ve made a considered decision that a significance level of 0.10 is appropriate (see what I wrote above), then I don’t see a problem with 0.103. Just be aware that you already have a relatively high chance of a false positive.

Finally, I hope that you didn’t choose the significance level based on the p-value that you obtained. You should choose the significance level before you conduct your study based on the pros and cons of Type 1 and Type II errors . Cherry picking a significance level based on your results can cause problems!

I hope this helps!

' src=

October 14, 2020 at 5:13 am

Hi Jim (if I may),

It’s been a while since I worked with many of the statistical concepts – reading through your brief guides has been really helpfull! I think this really helps to be back up to speed in no time.

– Martijn

October 14, 2020 at 8:25 pm

Hi Martijn,

I’m so happy to hear that my website has been helpful in get you up to speed! 🙂 You might consider my Introduction to Statistics ebook (and now in print) for an even more thorough introduction! A free sample is available in My Store .

Happy reading! Jim

' src=

September 25, 2020 at 1:53 am

Thanks for the reply Just would like to know if we have p value greater than 0.05. Lets say for example we have p equal to 0.35. In this case we fail to reject null hypothesis. Is this means, we failed to reject null hypothesis with (1-0.35) = 65% confidence interval? Is 65% confidence interval significant?

September 25, 2020 at 4:22 pm

Hi, as I mentioned, the confidence level is something that you set at the beginning of the study when you determine what significance level you will use. You do not change the confidence level based on your results.

In your example, you’re choosing a significance level of 0.05, which corresponds to using a confidence level of 95%. Those values are now fixed for your study. You don’t change them based on the results.

Then you analyze the data and your p-value of 0.35 is not significant. If you look at the CI with a confidence level of 95%, you’ll notice that it contains the null hypothesis value for your test. When the CI contains the null hypothesis value, that’s another way of determining that your results are not significant. If you use the corresponding p-values and CIs, those results will always agree. Read my article about confidence intervals to learn about that.

You don’t determine the confidence level at which you failed to reject the null. Just report the exact p-value with your findings to present that type of information.

September 23, 2020 at 11:38 pm

Hello , In case when P0.05, we fail to reject null hypothesis. So what will be the confidence level? and what are the chances of getting opposite results?

September 24, 2020 at 10:34 pm

Hi Tulajaram,

You set the confidence level so it equals 1 – significance level. So, if you use a significance level of 0.05, then you use a confidence level of 1 – 0.05 = 0.95. In this way, the confidence level results will match your hypothesis test results.

I’m not sure what you mean by “getting opposite results”?

' src=

June 20, 2020 at 9:34 am

Thank you for your educative piece on significance level in Statistics. Please comment on my question: I understand (I hope rightly so) that the 1% level relative to the 5% significance level ‘is a more stricter level of significance’ and hence allows very little room for error to be made in committing a Type I error. Would it therefore be right to conclude that if you fail to reject the null hypothesis at the 1% level of significance then you always won’t reject it at the 5% level of significance too? If not, please elaborate how the chosen significance levels (1%, 5% and 10%) relate to each other in rejecting or failing to reject the null hypothesis? Many thanks

June 20, 2020 at 9:05 pm

Yes, you’re correct that a 1% level is stricter. The significance level is the Type I error rate. So, a lower significance level (e.g., 1%) has, by definition, a lower Type I error rate. And, yes, it is possible to reject at one level, say 5%, and not reject at a lower level (1%). I show an example of this in my post about p-values and significance levels . It’s important to choose your significance level before conducting the study and then stick with it. Don’t change the significance level to obtain significant results.

' src=

June 14, 2020 at 10:26 am

Hi,thanks for the wonderful content.Would appreciate if you help me understand better.Imagining (tried to avoid the scary word hypothetical)a scenario of p being 0.07 and @ at 0.05. Will that still mean :”If the medicine has no effect in the population as a whole, 7 % of studies will obtain the effect observed in your sample, or larger, because of random sample error?”.And in that case what would be the extrapolated (although non advisable) error rate similar to the example quoted below?? While the exact error rate varies based on different assumptions, the values below use run-of-the-mill assumptions………. Regards

June 15, 2020 at 10:22 pm

Yes, that would be the correct way interpret a p-value of 0.07.

As for the error rate based on run of the mill assumptions, I don’t know for sure. I don’t have the article handy. But, I’d guess around 30%. However, it really depends on the prevalence of true effects. That’s essentially the overall probability of the alternative being true at the beginning of your study. And, that’s hard to say, but you can look at the significance of similar studies. But, you’d need to know how many were significant and not significant. Usually, we only hear about the significant studies. Bayesian statistics uses that approach. If your p-value is 0.07 but the alternative hypothesis is unlikely, the error rate could be much higher. So, take it with a grain of salt. The key point is that a p-value of near 0.05 (plus a little bit or minus a little bit) really is not strong evidence. 0.07 is a bit weaker. You really shouldn’t try to take it much further than that.

February 24, 2020 at 11:46 pm

Interesting conversations on the choice of significance level.

May I introduce my paper: Choosing the level of significance: decision theoretic approach

https://onlinelibrary.wiley.com/doi/abs/10.1111/abac.12172 https://papers.ssrn.com/sol3/papers.cfm?abstract_id=2652773

I also have R package OptSig, freely available from

https://cran.r-project.org/web/packages/OptSig/index.html

You may choose 1%, 5% or 10% level based on risk and power, but these levels have no theoretical justification at all and still completely arbitrary. My paper proposes that the optimal level be obtained by minimizing expected loss.

' src=

February 14, 2020 at 4:08 pm

When you say changing your alpha from .05 to .10 decreases the required evidence, that is literally true. It decreases the sample size required in order to detect the difference as significant; and/or increases the power of the study to detect this difference as statistically significant. I like to think in terms of all three: power, alpha, and sample size related to your effect size. The free software G*Power does a good job of determining these. That software also lets you graph, for example, statistical power as a function of sample size, but it isn’t always a smooth plot even though the axes are continuously and evenly scaled. For some statistical tests, that line is jerky. I think it has something to do with the shape of the distribution curve of something used in the calculation, but I’m embarrassed to say that I can’t recall what that is.

' src=

February 12, 2020 at 4:45 am

Hi Jim, If the concept of choosing the confidence level flexibly is integrated into six sigma, that will be even better and will lead to higher refinement of the outcome. Hope I am thinking in the right direction.

' src=

February 10, 2020 at 9:07 am

I consider “detectable difference” when designing a hypothesis study for inference.

If p is greater than 0.10, any difference has been overwhelmed by the noise.

If p is smaller than 0.10, there is a potential difference.

If p is smaller than 0.01, there is a detectable difference that will convince a skeptic.

February 10, 2020 at 10:14 am

As always, it’s great to hear from you!

In terms of p-values and the strength of the evidence they represent against the null hypothesis, I generally agree with your summary.

Personally, I consider p-values around 0.05 to represent a potential difference. The strength of evidence from a single study near 0.05 isn’t particularly strong by itself–but it probably warrants follow up. That’s based on simulation studies, which I discuss towards the end of my post about interpreting p-values and a different post about an empirical reproducibility study .

I agree that for p-values less than 0.01, the evidence is getting stronger. While a very low p-value doesn’t guarantee that the effect size is practically significant , it is pretty strong evidence.

' src=

February 10, 2020 at 12:56 am

I think there is something wrong in the Type 2 error. Rejecting a true null hypothesis is type 1 error will be a better explanation. Correct me if i am wrong.

February 10, 2020 at 1:26 am

Hi Ratnadeep, Yes, you’re entirely correct! Thanks for pointing that out. I accidentally flipped the numbers around. I’ve made the correction.

Comments and Questions Cancel reply

how does sample size influence hypothesis testing results

Power and Sample Size Determination

  •   1  
  • |   2  
  • |   3  
  • |   4  
  • |   5  
  • |   6  
  • |   7  
  • |   8  
  • |   9  
  • |   10  
  • |   11  

On This Page sidebar

Issues in Estimating Sample Size for Hypothesis Testing

Ensuring that a test has high power.

Learn More sidebar

All Modules

In the module on hypothesis testing for means and proportions, we introduced techniques for means, proportions, differences in means, and differences in proportions. While each test involved details that were specific to the outcome of interest (e.g., continuous or dichotomous) and to the number of comparison groups (one, two, more than two), there were common elements to each test. For example, in each test of hypothesis, there are two errors that can be committed. The first is called a Type I error and refers to the situation where we incorrectly reject H 0 when in fact it is true.   In the first step of any test of hypothesis, we select a level of significance, α , and α = P(Type I error) = P(Reject H 0 | H 0 is true). Because we purposely select a small value for α , we control the probability of committing a Type I error. The second type of error is called a Type II error and it is defined as the probability we do not reject H 0 when it is false. The probability of a Type II error is denoted β , and β =P(Type II error) = P(Do not Reject H 0 | H 0 is false). In hypothesis testing, we usually focus on power, which is defined as the probability that we reject H 0 when it is false, i.e., power = 1- β = P(Reject H 0 | H 0 is false). Power is the probability that a test correctly rejects a false null hypothesis. A good test is one with low probability of committing a Type I error (i.e., small α ) and high power (i.e., small β, high power).  

Here we present formulas to determine the sample size required to ensure that a test has high power. The sample size computations depend on the level of significance, aα, the desired power of the test (equivalent to 1-β), the variability of the outcome, and the effect size. The effect size is the difference in the parameter of interest that represents a clinically meaningful difference. Similar to the margin of error in confidence interval applications, the effect size is determined based on clinical or practical criteria and not statistical criteria.  

The concept of statistical power can be difficult to grasp. Before presenting the formulas to determine the sample sizes required to ensure high power in a test, we will first discuss power from a conceptual point of view.  

Suppose we want to test the following hypotheses at aα=0.05:  H 0 : μ = 90 versus H 1 : μ ≠ 90. To test the hypotheses, suppose we select a sample of size n=100. For this example, assume that the standard deviation of the outcome is σ=20. We compute the sample mean and then must decide whether the sample mean provides evidence to support the alternative hypothesis or not. This is done by computing a test statistic and comparing the test statistic to an appropriate critical value. If the null hypothesis is true (μ=90), then we are likely to select a sample whose mean is close in value to 90. However, it is also possible to select a sample whose mean is much larger or much smaller than 90. Recall from the Central Limit Theorem (see page 11 in the module on Probability ), that for large n (here n=100 is sufficiently large), the distribution of the sample means is approximately normal with a mean of

If the null hypothesis is true, it is possible to observe any sample mean shown in the figure below; all are possible under H 0 : μ = 90.  

Normal distribution of X when the mean of X is 90. A bell-shaped curve with a value of X-90 at the center.

Rejection Region for Test H 0 : μ = 90 versus H 1 : μ ≠ 90 at α =0.05

Standard normal distribution showing a mean of 90. The rejection areas are in the two tails at the extremes above and below the mean. If the alpha level is 0.05, then each tail accounts for an arean of 0.025.

The areas in the two tails of the curve represent the probability of a Type I Error, α= 0.05. This concept was discussed in the module on Hypothesis Testing .  

Now, suppose that the alternative hypothesis, H 1 , is true (i.e., μ ≠ 90) and that the true mean is actually 94. The figure below shows the distributions of the sample mean under the null and alternative hypotheses.The values of the sample mean are shown along the horizontal axis.  

Two overlapping normal distributions, one depicting the null hypothesis with a mean of 90 and the other showing the alternative hypothesis with a mean of 94. A more complete explanation of the figure is provided in the text below the figure.

If the true mean is 94, then the alternative hypothesis is true. In our test, we selected α = 0.05 and reject H 0 if the observed sample mean exceeds 93.92 (focusing on the upper tail of the rejection region for now). The critical value (93.92) is indicated by the vertical line. The probability of a Type II error is denoted β, and β = P(Do not Reject H 0 | H 0 is false), i.e., the probability of not rejecting the null hypothesis if the null hypothesis were true. β is shown in the figure above as the area under the rightmost curve (H 1 ) to the left of the vertical line (where we do not reject H 0 ). Power is defined as 1- β = P(Reject H 0 | H 0 is false) and is shown in the figure as the area under the rightmost curve (H 1 ) to the right of the vertical line (where we reject H 0 ).  

Note that β and power are related to α, the variability of the outcome and the effect size. From the figure above we can see what happens to β and power if we increase α. Suppose, for example, we increase α to α=0.10.The upper critical value would be 92.56 instead of 93.92. The vertical line would shift to the left, increasing α, decreasing β and increasing power. While a better test is one with higher power, it is not advisable to increase α as a means to increase power. Nonetheless, there is a direct relationship between α and power (as α increases, so does power).

β and power are also related to the variability of the outcome and to the effect size. The effect size is the difference in the parameter of interest (e.g., μ) that represents a clinically meaningful difference. The figure above graphically displays α, β, and power when the difference in the mean under the null as compared to the alternative hypothesis is 4 units (i.e., 90 versus 94). The figure below shows the same components for the situation where the mean under the alternative hypothesis is 98.

Overlapping bell-shaped distributions - one with a mean of 90 and the other with a mean of 98

Notice that there is much higher power when there is a larger difference between the mean under H 0 as compared to H 1 (i.e., 90 versus 98). A statistical test is much more likely to reject the null hypothesis in favor of the alternative if the true mean is 98 than if the true mean is 94. Notice also in this case that there is little overlap in the distributions under the null and alternative hypotheses. If a sample mean of 97 or higher is observed it is very unlikely that it came from a distribution whose mean is 90. In the previous figure for H 0 : μ = 90 and H 1 : μ = 94, if we observed a sample mean of 93, for example, it would not be as clear as to whether it came from a distribution whose mean is 90 or one whose mean is 94.

In designing studies most people consider power of 80% or 90% (just as we generally use 95% as the confidence level for confidence interval estimates). The inputs for the sample size formulas include the desired power, the level of significance and the effect size. The effect size is selected to represent a clinically meaningful or practically important difference in the parameter of interest, as we will illustrate.  

The formulas we present below produce the minimum sample size to ensure that the test of hypothesis will have a specified probability of rejecting the null hypothesis when it is false (i.e., a specified power). In planning studies, investigators again must account for attrition or loss to follow-up. The formulas shown below produce the number of participants needed with complete data, and we will illustrate how attrition is addressed in planning studies.

return to top | previous page | next page

Content ©2020. All Rights Reserved. Date last modified: March 13, 2020. Wayne W. LaMorte, MD, PhD, MPH

  • Find Us On Facebook
  • Follow on Twitter
  • Subscribe using RSS

The Importance and Effect of Sample Size

When conducting research about your customers, patients or products it’s usually impossible, or at least impractical, to collect data from all of the people or items that you are interested in. Instead, we take a sample (or subset) of the population of interest and learn what we can from that sample about the population.

There are lots of things that can affect how well our sample reflects the population and therefore how valid and reliable our conclusions will be. In this blog, we introduce some of the key concepts that should be considered when conducting a survey, including confidence levels and margins of error , power and effect sizes . (See the glossary below for some handy definitions of these terms.) Crucially, we’ll see that all of these are affected by how large a sample you take, i.e., the sample size .

Confidence and Margin of Error

Let’s start by considering an example where we simply want to estimate a characteristic of our population, and see the effect that our sample size has on how precise our estimate is.

The size of our sample dictates the amount of information we have and therefore, in part, determines our precision or level of confidence that we have in our sample estimates. An estimate always has an associated level of uncertainty, which depends upon the underlying variability of the data as well as the sample size. The more variable the population, the greater the uncertainty in our estimate. Similarly, the larger the sample size the more information we have and so our uncertainty reduces.

Suppose that we want to estimate the proportion of adults who own a smartphone in the UK. We could take a sample of 100 people and ask them. Note: it’s important to consider how the sample is selected to make sure that it is unbiased and representative of the population – we’ll blog on this topic another time.

The larger the sample size the more information we have and so our uncertainty reduces.

If 59 out of the 100 people own a smartphone, we estimate that the proportion in the UK is 59/100=59%. We can also construct an interval around this point estimate to express our uncertainty in it, i.e., our margin of error . For example, a 95% confidence interval for our estimate based on our sample of size 100 ranges from 49.36% to 68.64% (which can be calculated using our free online calculator ). Alternatively, we can express this interval by saying that our estimate is 59% with a margin of error of ±9.64%. This is a 95% confidence interval, which means that there is 95% probability that this interval contains the true proportion. In other words, if we were to collect 100 different samples from the population the true proportion would fall within this interval approximately 95 out of 100 times.

What would happen if we were to increase our sample size by going out and asking more people?

Suppose we ask another 900 people and find that, overall, 590 out of the 1000 people own a smartphone. Our estimate of the prevalence in the whole population is again 590/1000=59%. However, our confidence interval for the estimate has now narrowed considerably to 55.95% to 62.05%, a margin of error of ±3.05% – see Figure 1 below. Because we have more data and therefore more information, our estimate is more precise.

Precision versus sample size

As our sample size increases, the confidence in our estimate increases, our uncertainty decreases and we have greater precision. This is clearly demonstrated by the narrowing of the confidence intervals in the figure above. If we took this to the limit and sampled our whole population of interest then we would obtain the true value that we are trying to estimate – the actual proportion of adults who own a smartphone in the UK and we would have no uncertainty in our estimate.

Power and Effect Size

Increasing our sample size can also give us greater power to detect differences. Suppose in the example above that we were also interested in whether there is a difference in the proportion of men and women who own a smartphone.

We can estimate the sample proportions for men and women separately and then calculate the difference. When we sampled 100 people originally, suppose that these were made up of 50 men and 50 women, 25 and 34 of whom own a smartphone, respectively. So, the proportion of men and women owning smartphones in our sample is 25/50=50% and 34/50=68%, with less men than women owning a smartphone. The difference between these two proportions is known as the observed effect size. In this case, we observe that the gender effect is to reduce the proportion by 18% for men relative to women.

Is this observed effect significant, given such a small sample from the population, or might the proportions for men and women be the same and the observed effect due merely to chance?

We can use a statistical test to investigate this and, in this case, we use what’s known as the ‘Binomial test of equal proportions’ or ‘ two proportion z-test ‘. We find that there is insufficient evidence to establish a difference between men and women and the result is not considered statistically significant. The probability of observing a gender effect of 18% or more if there were truly no difference between men and women is greater than 5%, i.e., relatively likely and so the data provides no real evidence to suggest that the true proportions of men and women with smartphones are different. This cut-off of 5% is commonly used and is called the “ significance level ” of the test. It is chosen in advance of performing a test and is the probability of a type I error, i.e., of finding a statistically significant result, given that there is in fact no difference in the population.

What happens if we increase our sample size and include the additional 900 people in our sample?

Suppose that overall these were made up of 500 women and 500 men, 250 and 340 of whom own a smartphone, respectively. We now have estimates of 250/500=50% and 340/500=68% of men and women owning a smartphone. The effect size, i.e., the difference between the proportions, is the same as before (50% – 68% = ‑18%), but crucially we have more data to support this estimate of the difference. Using the statistical test of equal proportions again, we find that the result is statistically significant at the 5% significance level. Increasing our sample size has increased the power that we have to detect the difference in the proportion of men and women that own a smartphone in the UK.

Figure 2 provides a plot indicating the observed proportions of men and women, together with the associated 95% confidence intervals. We can clearly see that as our sample size increases the confidence intervals for our estimates for men and women narrow considerably. With a sample size of only 100, the confidence intervals overlap, offering little evidence to suggest that the proportions for men and women are truly any different. On the other hand, with the larger sample size of 1000 there is a clear gap between the two intervals and strong evidence to suggest that the proportions of men and women really are different.

The Binomial test above is essentially looking at how much these pairs of intervals overlap and if the overlap is small enough then we conclude that there really is a difference. (Note: The data in this blog are only for illustration; see this article for the results of a real survey on smartphone usage from earlier this year.)

Difference versus sample size

If your effect size is small then you will need a large sample size in order to detect the difference otherwise the effect will be masked by the randomness in your samples. Essentially, any difference will be well within the associated confidence intervals and you won’t be able to detect it. The ability to detect a particular effect size is known as statistical power . More formally, statistical power is the probability of finding a statistically significant result, given that there really is a difference (or effect) in the population. See our recent blog post “ Depression in Men ‘Regularly Ignored ‘” for another example of the effect of sample size on the likelihood of finding a statistically significant result.

So, larger sample sizes give more reliable results with greater precision and power, but they also cost more time and money. That’s why you should always perform a sample size calculation before conducting a survey to ensure that you have a sufficiently large sample size to be able to draw meaningful conclusions, without wasting resources on sampling more than you really need. We’ve put together some free, online statistical calculators to help you carry out some statistical calculations of your own, including sample size calculations for estimating a proportion and comparing two proportions .

Margin of error – This is the level of precision you require. It is the range in which the value that you are trying to measure is estimated to be and is often expressed in percentage points (e.g., ±2%). A narrower margin of error requires a larger sample size.

Confidence level – This conveys the amount of uncertainty associated with an estimate. It is the chance that the confidence interval (margin of error around the estimate) will contain the true value that you are trying to estimate. A higher confidence level requires a larger sample size.

Power – This is the probability that we find statistically significant evidence of a difference between the groups, given that there is a difference in the population. A greater power requires a larger sample size.

Effect size – This is the estimated difference between the groups that we observe in our sample. To detect a difference with a specified power, a smaller effect size will require a larger sample size.

Related Articles

  • “Modest” but “statistically significant”…what does that mean? (statsoft.com)
  • Legal vs clinical trials: An explanation of sampling errors and sample size (statslife.org.uk)

Tell us what you want to achieve

  • Data Collection & Management
  • Data Mining
  • Innovation & Research
  • Qualitative Analysis
  • Surveys & Sampling
  • Visualisation
  • Agriculture
  • Environment
  • Market Research
  • Public Sector

Select Statistical Services Ltd

Oxygen House, Grenadier Road, Exeter Business Park,

Exeter EX1 3LH

t: 01392 440426

e: [email protected]

Sign up to our Newsletter

  • Please tick this box to confirm that you are happy for us to store and process the information supplied above for the purpose of managing your subscription to our newsletter.
  • Email This field is for validation purposes and should be left unchanged.
  • Telephone Number

' width=

  • By using this form you agree with the storage and handling of your data by this website.
  • Comments This field is for validation purposes and should be left unchanged.

Enquiry - Jobs

' width=

  • Name This field is for validation purposes and should be left unchanged.

  • Foundations
  • Write Paper

Search form

  • Experiments
  • Anthropology
  • Self-Esteem
  • Social Anxiety
  • Statistics >
  • Statistical Significance

Statistical Significance And Sample Size

Comparing statistical significance, sample size and expected effects are important before constructing and experiment.

This article is a part of the guide:

  • Significance 2
  • Experimental Probability
  • Cronbach’s Alpha
  • Systematic Error
  • Random Error

Browse Full Outline

  • 1 Inferential Statistics
  • 2.1 Bayesian Probability
  • 3.1.1 Significance 2
  • 3.2 Significant Results
  • 3.3 Sample Size
  • 3.4 Margin of Error
  • 3.5.1 Random Error
  • 3.5.2 Systematic Error
  • 3.5.3 Data Dredging
  • 3.5.4 Ad Hoc Analysis
  • 3.5.5 Regression Toward the Mean
  • 4.1 P-Value
  • 4.2 Effect Size
  • 5.1 Philosophy of Statistics
  • 6.1.1 Reliability 2
  • 6.2 Cronbach’s Alpha

A power analysis is used to reveal the minimum sample size which is required compared to the significance level and expected effects.

Many effects have been missed due to the lack of planning a study and thus having a too low sample size. Also, there is nothing wrong with having a too big sample size, but often much money and efforts are required to increase the sample size, and it could prove to be unnecessary.

how does sample size influence hypothesis testing results

Generalization

If you want to generalize the findings of your research on a small sample to a whole population, your sample size should at least be of a size that could meet the significance level, given the expected effects. Expected effects are often worked out from pilot studies , common sense-thinking or by comparing similar experiments. Expected effects may not be fully accurate.

Sample

Comparing the statistical significance and sample size is done to be able to extend the results obtained for the given sample to the whole population.

It is useful to do this before running the experiment - sometimes you may find that you need a much bigger sample size to get a significant result , than it is feasible to obtain (thus making you rethink before going through the whole procedure).

Different experiments invariably have different sample sizes and significance levels. The concepts are very useful in biological , economical and social experiments and all kinds of generalizations based on information about a smaller subset.

how does sample size influence hypothesis testing results

The results of your experiment are validated and can be accepted only if the results for the given experiment pass a significance test . The sample size is adjusted using statistical power .

For example, if an experimenter takes a survey of a group of 100 people and decides the presidential votes based on this data, the results are likely to be highly erroneous because the population size is huge compared to the sample size.

The needed effect is much smaller since this experiment requires much 'power'.

Confidence Level

The sample size depends on the confidence interval and confidence level. The lower the confidence interval required, the higher sample size is needed.

For example, if you are interviewing 1000 people in a town on their choice of presidential candidate, your results may be accurate to within +/- 4% of your findings. If you wish to lower the confidence interval to +/- 1%, then you will naturally need to interview more people, which means an increase in the sample size.

If you want your presidential results to be of 99% confidence level instead of 95%, then you will need to have a much higher sample size of people to interview. This means that the survey needs higher power to accept a hypothesis .

Increasing the Sample Size

Some researchers choose to increase their sample size if they have an effect which is almost within significance level. This is done since the researcher suspects that he is short of samples, rather than that there is no effect there. You need to be careful using this method, as it increases the chances of creating a false positive result .

When you have a higher sample size, the likelihood of encountering Type-I and Type-II errors occurring reduces, at least if other parts of your study is carefully constructed and problems avoided. Higher sample size allows the researcher to increase the significance level of the findings, since the confidence of the result are likely to increase with a higher sample size. This is to be expected because larger the sample size, the more accurately it is expected to mirror the behavior of the whole group.

Therefore if you want to reject your null hypothesis , then you should make sure your sample size is at least equal to the sample size needed for the statistical significance chosen and expected effects.

  • Psychology 101
  • Flags and Countries
  • Capitals and Countries

Siddharth Kalla (Jun 18, 2009). Statistical Significance And Sample Size. Retrieved Sep 07, 2024 from Explorable.com: https://explorable.com/statistical-significance-sample-size

You Are Allowed To Copy The Text

The text in this article is licensed under the Creative Commons-License Attribution 4.0 International (CC BY 4.0) .

This means you're free to copy, share and adapt any parts (or all) of the text in the article, as long as you give appropriate credit and provide a link/reference to this page.

That is it. You don't need our permission to copy the article; just include a link/reference back to this page. You can use it freely (with some kind of link), and we're also okay with people reprinting in publications like books, blogs, newsletters, course-material, papers, wikipedia and presentations (with clear attribution).

Want to stay up to date? Follow us!

Save this course for later.

Don't have time for it all now? No problem, save it as a course and come back to it later.

Footer bottom

  • Privacy Policy

how does sample size influence hypothesis testing results

  • Subscribe to our RSS Feed
  • Like us on Facebook
  • Follow us on Twitter

StatAnalytica

Step-by-step guide to hypothesis testing in statistics

hypothesis testing in statistics

Hypothesis testing in statistics helps us use data to make informed decisions. It starts with an assumption or guess about a group or population—something we believe might be true. We then collect sample data to check if there is enough evidence to support or reject that guess. This method is useful in many fields, like science, business, and healthcare, where decisions need to be based on facts.

Learning how to do hypothesis testing in statistics step-by-step can help you better understand data and make smarter choices, even when things are uncertain. This guide will take you through each step, from creating your hypothesis to making sense of the results, so you can see how it works in practical situations.

What is Hypothesis Testing?

Table of Contents

Hypothesis testing is a method for determining whether data supports a certain idea or assumption about a larger group. It starts by making a guess, like an average or a proportion, and then uses a small sample of data to see if that guess seems true or not.

For example, if a company wants to know if its new product is more popular than its old one, it can use hypothesis testing. They start with a statement like “The new product is not more popular than the old one” (this is the null hypothesis) and compare it with “The new product is more popular” (this is the alternative hypothesis). Then, they look at customer feedback to see if there’s enough evidence to reject the first statement and support the second one.

Simply put, hypothesis testing is a way to use data to help make decisions and understand what the data is really telling us, even when we don’t have all the answers.

Importance Of Hypothesis Testing In Decision-Making And Data Analysis

Hypothesis testing is important because it helps us make smart choices and understand data better. Here’s why it’s useful:

  • Reduces Guesswork : It helps us see if our guesses or ideas are likely correct, even when we don’t have all the details.
  • Uses Real Data : Instead of just guessing, it checks if our ideas match up with real data, which makes our decisions more reliable.
  • Avoids Errors : It helps us avoid mistakes by carefully checking if our ideas are right so we don’t make costly errors.
  • Shows What to Do Next : It tells us if our ideas work or not, helping us decide whether to keep, change, or drop something. For example, a company might test a new ad and decide what to do based on the results.
  • Confirms Research Findings : It makes sure that research results are accurate and not just random chance so that we can trust the findings.

Here’s a simple guide to understanding hypothesis testing, with an example:

1. Set Up Your Hypotheses

Explanation: Start by defining two statements:

  • Null Hypothesis (H0): This is the idea that there is no change or effect. It’s what you assume is true.
  • Alternative Hypothesis (H1): This is what you want to test. It suggests there is a change or effect.

Example: Suppose a company says their new batteries last an average of 500 hours. To check this:

  • Null Hypothesis (H0): The average battery life is 500 hours.
  • Alternative Hypothesis (H1): The average battery life is not 500 hours.

2. Choose the Test

Explanation: Pick a statistical test that fits your data and your hypotheses. Different tests are used for various kinds of data.

Example: Since you’re comparing the average battery life, you use a one-sample t-test .

3. Set the Significance Level

Explanation: Decide how much risk you’re willing to take if you make a wrong decision. This is called the significance level, often set at 0.05 or 5%.

Example: You choose a significance level of 0.05, meaning you’re okay with a 5% chance of being wrong.

4. Gather and Analyze Data

Explanation: Collect your data and perform the test. Calculate the test statistic to see how far your sample result is from what you assumed.

Example: You test 30 batteries and find they last an average of 485 hours. You then calculate how this average compares to the claimed 500 hours using the t-test.

5. Find the p-Value

Explanation: The p-value tells you the probability of getting a result as extreme as yours if the null hypothesis is true.

Example: You find a p-value of 0.0001. This means there’s a very small chance (0.01%) of getting an average battery life of 485 hours or less if the true average is 500 hours.

6. Make Your Decision

Explanation: Compare the p-value to your significance level. If the p-value is smaller, you reject the null hypothesis. If it’s larger, you do not reject it.

Example: Since 0.0001 is much less than 0.05, you reject the null hypothesis. This means the data suggests the average battery life is different from 500 hours.

7. Report Your Findings

Explanation: Summarize what the results mean. State whether you rejected the null hypothesis and what that implies.

Example: You conclude that the average battery life is likely different from 500 hours. This suggests the company’s claim might not be accurate.

Hypothesis testing is a way to use data to check if your guesses or assumptions are likely true. By following these steps—setting up your hypotheses, choosing the right test, deciding on a significance level, analyzing your data, finding the p-value, making a decision, and reporting results—you can determine if your data supports or challenges your initial idea.

Understanding Hypothesis Testing: A Simple Explanation

Hypothesis testing is a way to use data to make decisions. Here’s a straightforward guide:

1. What is the Null and Alternative Hypotheses?

  • Null Hypothesis (H0): This is your starting assumption. It says that nothing has changed or that there is no effect. It’s what you assume to be true until your data shows otherwise. Example: If a company says their batteries last 500 hours, the null hypothesis is: “The average battery life is 500 hours.” This means you think the claim is correct unless you find evidence to prove otherwise.
  • Alternative Hypothesis (H1): This is what you want to find out. It suggests that there is an effect or a difference. It’s what you are testing to see if it might be true. Example: To test the company’s claim, you might say: “The average battery life is not 500 hours.” This means you think the average battery life might be different from what the company says.

2. One-Tailed vs. Two-Tailed Tests

  • One-Tailed Test: This test checks for an effect in only one direction. You use it when you’re only interested in finding out if something is either more or less than a specific value. Example: If you think the battery lasts longer than 500 hours, you would use a one-tailed test to see if the battery life is significantly more than 500 hours.
  • Two-Tailed Test: This test checks for an effect in both directions. Use this when you want to see if something is different from a specific value, whether it’s more or less. Example: If you want to see if the battery life is different from 500 hours, whether it’s more or less, you would use a two-tailed test. This checks for any significant difference, regardless of the direction.

3. Common Misunderstandings

  • Clarification: Hypothesis testing doesn’t prove that the null hypothesis is true. It just helps you decide if you should reject it. If there isn’t enough evidence against it, you don’t reject it, but that doesn’t mean it’s definitely true.
  • Clarification: A small p-value shows that your data is unlikely if the null hypothesis is true. It suggests that the alternative hypothesis might be right, but it doesn’t prove the null hypothesis is false.
  • Clarification: The significance level (alpha) is a set threshold, like 0.05, that helps you decide how much risk you’re willing to take for making a wrong decision. It should be chosen carefully, not randomly.
  • Clarification: Hypothesis testing helps you make decisions based on data, but it doesn’t guarantee your results are correct. The quality of your data and the right choice of test affect how reliable your results are.

Benefits and Limitations of Hypothesis Testing

  • Clear Decisions: Hypothesis testing helps you make clear decisions based on data. It shows whether the evidence supports or goes against your initial idea.
  • Objective Analysis: It relies on data rather than personal opinions, so your decisions are based on facts rather than feelings.
  • Concrete Numbers: You get specific numbers, like p-values, to understand how strong the evidence is against your idea.
  • Control Risk: You can set a risk level (alpha level) to manage the chance of making an error, which helps avoid incorrect conclusions.
  • Widely Used: It can be used in many areas, from science and business to social studies and engineering, making it a versatile tool.

Limitations

  • Sample Size Matters: The results can be affected by the size of the sample. Small samples might give unreliable results, while large samples might find differences that aren’t meaningful in real life.
  • Risk of Misinterpretation: A small p-value means the results are unlikely if the null hypothesis is true, but it doesn’t show how important the effect is.
  • Needs Assumptions: Hypothesis testing requires certain conditions, like data being normally distributed . If these aren’t met, the results might not be accurate.
  • Simple Decisions: It often results in a basic yes or no decision without giving detailed information about the size or impact of the effect.
  • Can Be Misused: Sometimes, people misuse hypothesis testing, tweaking data to get a desired result or focusing only on whether the result is statistically significant.
  • No Absolute Proof: Hypothesis testing doesn’t prove that your hypothesis is true. It only helps you decide if there’s enough evidence to reject the null hypothesis, so the conclusions are based on likelihood, not certainty.

Final Thoughts 

Hypothesis testing helps you make decisions based on data. It involves setting up your initial idea, picking a significance level, doing the test, and looking at the results. By following these steps, you can make sure your conclusions are based on solid information, not just guesses.

This approach lets you see if the evidence supports or contradicts your initial idea, helping you make better decisions. But remember that hypothesis testing isn’t perfect. Things like sample size and assumptions can affect the results, so it’s important to be aware of these limitations.

In simple terms, using a step-by-step guide for hypothesis testing is a great way to better understand your data. Follow the steps carefully and keep in mind the method’s limits.

What is the difference between one-tailed and two-tailed tests?

 A one-tailed test assesses the probability of the observed data in one direction (either greater than or less than a certain value). In contrast, a two-tailed test looks at both directions (greater than and less than) to detect any significant deviation from the null hypothesis.

How do you choose the appropriate test for hypothesis testing?

The choice of test depends on the type of data you have and the hypotheses you are testing. Common tests include t-tests, chi-square tests, and ANOVA. You get more details about ANOVA, you may read Complete Details on What is ANOVA in Statistics ?  It’s important to match the test to the data characteristics and the research question.

What is the role of sample size in hypothesis testing?  

Sample size affects the reliability of hypothesis testing. Larger samples provide more reliable estimates and can detect smaller effects, while smaller samples may lead to less accurate results and reduced power.

Can hypothesis testing prove that a hypothesis is true?  

Hypothesis testing cannot prove that a hypothesis is true. It can only provide evidence to support or reject the null hypothesis. A result can indicate whether the data is consistent with the null hypothesis or not, but it does not prove the alternative hypothesis with certainty.

Related Posts

how-to-find-the=best-online-statistics-homework-help

How to Find the Best Online Statistics Homework Help

why-spss-homework-help-is-an-important-aspects-for-students

Why SPSS Homework Help Is An Important aspect for Students?

Leave a comment cancel reply.

Your email address will not be published. Required fields are marked *

  • Prompt Library
  • DS/AI Trends
  • Stats Tools
  • Interview Questions
  • Generative AI
  • Machine Learning
  • Deep Learning

Level of Significance & Hypothesis Testing

level of significance and hypothesis testing

In hypothesis testing , the level of significance is a measure of how confident you can be about rejecting the null hypothesis. This blog post will explore what hypothesis testing is and why understanding significance levels are important for your data science projects. In addition, you will also get to test your knowledge of level of significance towards the end of the blog with the help of quiz . These questions can help you test your understanding and prepare for data science / statistics interviews . Before we look into what level of significance is, let’s quickly understand what is hypothesis testing.

Table of Contents

What is Hypothesis testing and how is it related to significance level?

Hypothesis testing can be defined as tests performed to evaluate whether a claim or theory about something is true or otherwise. In order to perform hypothesis tests, the following steps need to be taken:

  • Hypothesis formulation: Formulate the null and alternate hypothesis
  • Data collection: Gather the sample of data
  • Statistical tests: Determine the statistical test and test statistics. The statistical tests can be z-test or t-test depending upon the number of data samples and/or whether the population variance is known otherwise.
  • Set the level of significance
  • Calculate the p-value
  • Draw conclusions: Based on the value of p-value and significance level, reject the null hypothesis or otherwise.

A detailed explanation is provided in one of my related posts titled hypothesis testing explained with examples .

What is the level of significance?

The level of significance is defined as the criteria or threshold value based on which one can reject the null hypothesis or fail to reject the null hypothesis. The level of significance determines whether the outcome of hypothesis testing is statistically significant or otherwise. The significance level is also called as alpha level.

Another way of looking at the level of significance is the value which represents the likelihood of making a type I error . You may recall that Type I error occurs while evaluating hypothesis testing outcomes. If you reject the null hypothesis by mistake, you end up making a Type I error. This scenario is also termed as “false positive”. Take an example of a person alleged with committing a crime. The null hypothesis is that the person is not guilty. Type I error happens when you reject the null hypothesis by mistake. Given the example, a Type I error happens when you reject the null hypothesis that the person is not guilty by mistake. The innocent person is convicted.

The level of significance can take values such as 0.1 , 0.05 , 0.01 . The most common value of the level of significance is 0.05 . The lower the value of significance level, the lesser is the chance of type I error. That would essentially mean that the experiment or hypothesis testing outcome would really need to be highly precise for one to reject the null hypothesis. The likelihood of making a type I error would be very low. However, that does increase the chances of making type II errors as you may make mistakes in failing to reject the null hypothesis. You may want to read more details in relation to type I errors and type II errors in this post – Type I errors and Type II errors in hypothesis testing

The outcome of the hypothesis testing is evaluated with the help of a p-value. If the p-value is less than the level of significance, then the hypothesis testing outcome is statistically significant. On the other hand, if the hypothesis testing outcome is not statistically significant or the p-value is more than the level of significance, then we fail to reject the null hypothesis. The same is represented in the picture below for a right-tailed test. I will be posting details on different types of tail test in future posts.

level of significance and hypothesis testing

The picture below represents the concept for two-tailed hypothesis test:

level of significance and two-tailed test

For example: Let’s say that a school principal wants to find out whether extra coaching of 2 hours after school help students do better in their exams. The hypothesis would be as follows:

  • Null hypothesis : There is no difference between the performance of students even after providing extra coaching of 2 hours after the schools are over.
  • Alternate hypothesis : Students perform better when they get extra coaching of 2 hours after the schools are over. This hypothesis testing example would require a level of significant value at 0.05 or simply put, it would need to be highly precise that there’s actually a difference between the performance of students based on whether they take extra coaching.

Now, let’s say that we conduct this experiment with 100 students and measure their scores in exams. The test statistics is computed to be z=-0.50 (p-value=0.62). Since the p-value is more than 0.05, we fail to reject the null hypothesis. There is not enough evidence to show that there’s a difference in the performance of students based on whether they get extra coaching.

While performing hypothesis tests or experiments, it is important to keep the level of significance in mind.

Why does one need a level of significance?

In hypothesis tests, if we do not have some sort of threshold by which to determine whether your results are statistically significant enough for you to reject the null hypothesis, then it would be tough for us to determine whether your findings are significant or not. This is why we take into account levels of significance when performing hypothesis tests and experiments.

Since hypothesis testing helps us in making decisions about our data, having a level of significance set up allows one to know what sort of chances their findings might have of actually being due to the null hypothesis. If you set your level of significance at 0.05 for example, it would mean that there’s only a five percent chance that the difference between groups (assuming two groups are tested) is due to random sampling error. So if we found a difference in the performance of students based on whether they take extra coaching, we would need to consider other factors that could have contributed to the difference.

This is why hypothesis testing and level of significance go hand in hand with one another: hypothesis tests help us know whether our data falls within a certain range where it’s statistically significant or not so statistically significant whereas the level of significance tells us how likely is it that our hypothesis testing results are not due to random sampling error.

How is the level of significance used in hypothesis testing?

The level of significance along with the test statistic and p-value formed a key part of hypothesis testing. The value that you derive from hypothesis testing depends on whether or not you accept/reject the null hypothesis, given your findings at each step. Before going into rejection vs non-rejection, let’s understand the terms better.

If the test statistic falls within the critical region, you reject the null hypothesis. This means that your findings are statistically significant and support the alternate hypothesis. The value of the p-value determines how likely it is for finding this outcome if, in fact, the null hypothesis were true. If the p-value is less than or equal to the level of significance, you reject the null hypothesis. This means that your hypothesis testing outcome was statistically significant at a certain degree and in favor of the alternate hypothesis.

If on the other hand, the p-value is greater than alpha level or significance level, then you fail to reject the null hypothesis. These findings are not statistically significant enough for one to reject the null hypothesis. The same is represented in the diagram below:

level of significance and p-value

Level of Significance – Quiz / Interview Questions

Here are some practice questions which can help you in testing your questions, and, also prepare for interviews.

#1. Which of the following will result in greater type I error?

#2. the p-value of 0.03 is statistically significant for significance level as 0.01, #3. the p-value less than the level of significance would mean which of the following, #4. the statistically significant outcome of hypothesis testing would mean which of the following, #5. which of the following is looks to be inappropriate level of significance, #6. which of the following will result in greater type ii error, #7. level of significance is also called as ________, #8. which one of the following is considered most popular choice of significance level, recent posts.

Ajitesh Kumar

  • ROC Curve & AUC Explained with Python Examples - August 28, 2024
  • Accuracy, Precision, Recall & F1-Score – Python Examples - August 28, 2024
  • Logistic Regression in Machine Learning: Python Example - August 26, 2024

Oops! Check your answers again. The minimum pass percentage is 70%.

Share your score!

Hypothesis testing is an important statistical concept that helps us determine whether the claim made about anything is true or otherwise. The hypothesis test statistic, level of significance, and p-value all work together to help you make decisions about your data. If our hypothesis tests show enough evidence to reject the null hypothesis, then we know statistically significant findings are at hand. This post gave you ideas for how you can use hypothesis testing in your experiments by understanding what it means when someone rejects or fails to reject the null hypothesis.

Ajitesh Kumar

3 responses.

Well explained with examples and helpful illustration

Thank you for your feedback

Well explained

Leave a Reply Cancel reply

Your email address will not be published. Required fields are marked *

  • Search for:

ChatGPT Prompts (250+)

  • Generate Design Ideas for App
  • Expand Feature Set of App
  • Create a User Journey Map for App
  • Generate Visual Design Ideas for App
  • Generate a List of Competitors for App
  • ROC Curve & AUC Explained with Python Examples
  • Accuracy, Precision, Recall & F1-Score – Python Examples
  • Logistic Regression in Machine Learning: Python Example
  • Reducing Overfitting vs Models Complexity: Machine Learning
  • Model Parallelism vs Data Parallelism: Examples

Data Science / AI Trends

  • • Prepend any arxiv.org link with talk2 to load the paper into a responsive chat application
  • • Custom LLM and AI Agents (RAG) On Structured + Unstructured Data - AI Brain For Your Organization
  • • Guides, papers, lecture, notebooks and resources for prompt engineering
  • • Common tricks to make LLMs efficient and stable
  • • Machine learning in finance

Free Online Tools

  • Create Scatter Plots Online for your Excel Data
  • Histogram / Frequency Distribution Creation Tool
  • Online Pie Chart Maker Tool
  • Z-test vs T-test Decision Tool
  • Independent samples t-test calculator

Recent Comments

I found it very helpful. However the differences are not too understandable for me

Very Nice Explaination. Thankyiu very much,

in your case E respresent Member or Oraganization which include on e or more peers?

Such a informative post. Keep it up

Thank you....for your support. you given a good solution for me.

U.S. flag

An official website of the United States government

The .gov means it’s official. Federal government websites often end in .gov or .mil. Before sharing sensitive information, make sure you’re on a federal government site.

The site is secure. The https:// ensures that you are connecting to the official website and that any information you provide is encrypted and transmitted securely.

  • Publications
  • Account settings

Preview improvements coming to the PMC website in October 2024. Learn More or Try it out now .

  • Advanced Search
  • Journal List
  • Clin Orthop Relat Res
  • v.466(9); 2008 Sep

Logo of corr

Statistics in Brief: The Importance of Sample Size in the Planning and Interpretation of Medical Research

David jean biau.

Département de Biostatistique et Informatique Médicale, INSERM – UMR-S 717, AP-HP, Université Paris 7, Hôpital Saint Louis, 1, avenue Claude-Vellefaux, Paris Cedex 10, 75475 France

Solen Kernéis

Raphaël porcher.

The increasing volume of research by the medical community often leads to increasing numbers of contradictory findings and conclusions. Although the differences observed may represent true differences, the results also may differ because of sampling variability as all studies are performed on a limited number of specimens or patients. When planning a study reporting differences among groups of patients or describing some variable in a single group, sample size should be considered because it allows the researcher to control for the risk of reporting a false-negative finding (Type II error) or to estimate the precision his or her experiment will yield. Equally important, readers of medical journals should understand sample size because such understanding is essential to interpret the relevance of a finding with regard to their own patients. At the time of planning, the investigator must establish (1) a justifiable level of statistical significance, (2) the chances of detecting a difference of given magnitude between the groups compared, ie, the power, (3) this targeted difference (ie, effect size), and (4) the variability of the data (for quantitative data). We believe correct planning of experiments is an ethical issue of concern to the entire community.

Introduction

“Statistical analysis allows us to put limits on our uncertainty, but not to prove anything.”— Douglas G. Altman [ 1 ]

The growing need for medical practice based on evidence has generated an increasing medical literature supported by statistics: readers expect and presume medical journals publish only studies with unquestionable results they can use in their everyday practice and editors expect and often request authors provide rigorously supportable answers. Researchers submit articles based on presumably valid outcome measures, analyses, and conclusions claiming or implying the superiority of one treatment over another, the usefulness of a new diagnostic test, or the prognostic value of some sign. Paradoxically, the increasing frequency of seemingly contradictory results may be generating increasing skepticism in the medical community.

One fundamental reason for this conundrum takes root in the theory of hypothesis testing developed by Pearson and Neyman in the late 1920s [ 24 , 25 ]. The majority of medical research is presented in the form of a comparison, the most obvious being treatment comparisons in randomized controlled trials. To assess whether the difference observed is likely attributable to chance alone or to a true difference, researchers set a null hypothesis that there is no difference between the alternative treatments. They then determine the probability (the p value), they could have obtained the difference observed or a larger difference if the null hypothesis were true; if this probability is below some predetermined explicit significance level, the null hypothesis (ie, there is no difference) is rejected. However, regardless of study results, there is always a chance to conclude there is a difference when in fact there is not (Type I error or false positive) or to report there is no difference when a true difference does exist (Type II error or false negative) and the study has simply failed to detect it (Table  1 ). The size of the sample studied is a major determinant of the risk of reporting false-negative findings. Therefore, sample size is important for planning and interpreting medical research.

Table 1

Type I and Type II errors during hypothesis testing

TruthStudy findings
Null hypothesis is not rejectedNull hypothesis is rejected
Null hypothesis is trueTrue negativeType I error (alpha) (False positive)
Null hypothesis is falseType II error (beta) (False negative)True positive

For that reason, we believe readers should be adequately informed of the frequent issues related to sample size, such as (1) the desired level of statistical significance, (2) the chances of detecting a difference of given magnitude between the groups compared, ie, the power, (3) this targeted difference, and (4) the variability of the data (for quantitative data). We will illustrate these matters with a comparison between two treatments in a surgical randomized controlled trial. The use of sample size also will be presented in other common areas of statistics, such as estimation and regression analyzes.

Desired Level of Significance

The level of statistical significance α corresponds to the probability of Type I error, namely, the probability of rejecting the null hypothesis of “no difference between the treatments compared” when in fact it is true. The decision to reject the null hypothesis is based on a comparison of the prespecified level of the test arbitrarily chosen with the test procedure’s p value. Controlling for Type I error is paramount to medical research to avoid the spread of new or perpetuation of old treatments that are ineffective. For the majority of hypothesis tests, the level of significance is arbitrarily chosen at 5%. When an investigator chooses α = 5%, if the test’s procedure p value computed is less than 5%, the null hypothesis will be rejected and the treatments compared will be assumed to be different.

To reduce the probability of Type I error, we may choose to reduce the level of statistical significance to 1% or less [ 29 ]. However, the level of statistical significance also influences the sample size calculation: the lower the chosen level of statistical significance, the larger the sample size will be, considering all other parameters remain the same (see example below and Appendix 1). Consequently, there are domains where higher levels of statistical significance are used so that the sample size remains restricted, such as for randomized Phase II screening designs in cancer [ 26 ]. We believe the choice of a significance level greater than 5% should be restricted to particular cases.

The power of a test is defined as 1 − the probability of Type II error. The Type II error is concluding at no difference (the null is not rejected) when in fact there is a difference, and its probability is named β. Therefore, the power of a study reflects the probability of detecting a difference when this difference exists. It is also very important to medical research that studies are planned with an adequate power so that meaningful conclusions can be issued if no statistical difference has been shown between the treatments compared. More power means less risk for Type II errors and more chances to detect a difference when it exists.

Power should be determined a priori to be at least 80% and preferably 90%. The latter means, if the true difference between treatments is equal to the one we planned, there is only 10% chance the study will not detect it. Sample size increases with increasing power (Fig.  1 ).

An external file that holds a picture, illustration, etc.
Object name is 11999_2008_346_Fig1_HTML.jpg

The graphs show the distribution of the test statistic (z-test) for the null hypothesis (plain line) and the alternative hypothesis (dotted line) for a sample size of ( A ) 32 patients per group, ( B ) 64 patients per group, and ( C ) 85 patients per group. For a difference in mean of 10, a standard deviation of 20, and a significance level α of 5%, the power (shaded area) increases from ( A ) 50%, to ( B ) 80%, and ( C ) 90%. It can be seen, as power increases, the test statistics yielded under the alternative hypothesis (there is a difference in the two comparison groups) are more likely to be greater than the critical value 1.96.

Very commonly, power calculations have not been performed before conducting the trial [ 3 , 8 ], and when facing nonsignificant results, investigators sometimes compute post hoc power analyses, also called observed power. For this purpose, investigators use the observed difference and variability and the sample size of the trial to determine the power they would have had to detect this particular difference. However, post hoc power analyses have little statistical meaning for three reasons [ 9 , 13 ]. First, because there is a one-to-one relationship between p values and post hoc power, the latter does not convey any additional information on the sample than the former. Second, nonsignificant p values always correspond to low power and post hoc power, at best, will be slightly larger than 50% for p values equal to or greater than 0.05. Third, when computing post hoc power, investigators implicitly make the assumption that the difference observed is clinically meaningful and more representative of the truth than the null hypothesis they precisely were not able to reject. However, in the theory of hypothesis testing, the difference observed should be used only to choose between the hypotheses stated a priori; a posteriori, the use of confidence intervals is preferable to judge the relevance of a finding. The confidence interval represents the range of values we can be confident to some extent includes the true difference. It is related directly to sample size and conveys more information than p values. Nonetheless, post hoc power analyses educate readers about the importance of considering sample size by explicitly raising the issue.

The Targeted Difference Between the Alternative Treatments

The targeted difference between the alternative treatments is determined a priori by the investigator, typically based on preliminary data. The larger the expected difference is, the smaller the required sample size will be. However, because the sample size based on the difference expected may be too large to achieve, investigators sometimes choose to power their trial to detect a difference larger than one would normally expect to reduce the sample size and minimize the time and resources dedicated to the trial. However, if the targeted difference between the alternative treatments is larger than the true difference, the trial may fail to conclude a difference between the two treatments when a smaller, and still meaningful, difference exists. This smallest meaningful difference sometimes is expressed as the “minimal clinically important difference,” namely, “the smallest difference in score in the domain of interest which patients perceive as beneficial and which would mandate, in the absence of troublesome side effects and excessive costs, a change in the patient’s management” [ 15 ]. Because theoretically the minimal clinically important difference is a multidimensional phenomenon that encompasses a wide range of complex issues of a particular treatment in a unique setting, it usually is determined by consensus among clinicians with expertise in the domain. When the measure of treatment effect is based on a score, researchers may use empiric definitions of clinically meaningful difference. For instance, Michener et al. [ 21 ], in a prospective study of 63 patients with various shoulder abnormalities, determined the minimal change perceived as clinically meaningful by the patients for the patient self-report section of the American Shoulder and Elbow Surgeons Standardized Shoulder Assessment Form was 6.7 points of 100 points. Similarly, Bijur et al. [ 5 ], in a prospective cohort study of 108 adults presenting to the emergency department with acute pain, determined the minimal change perceived as clinically meaningful by patients for acute pain measured on the visual analog scale was 1.4 points. There is no reason to try to detect a difference below the minimal clinically important difference because, even if it proves statistically significant, it will not be meaningful.

The meaningful clinically important difference should not be confused with the effect size. The effect size is a dimensionless measure of the magnitude of a relation between two or more variables, such as Cohen’s d standardized difference [ 6 ], but also odds ratio, Pearson’s r correlation coefficient, etc. Sometimes studies are planned to detect a particular effect size instead of being planned to detect a particular difference between the two treatments. According to Cohen [ 6 ], 0.2 is indicative of a small effect, 0.5 a medium effect, and 0.8 a large effect size. One of the advantages of doing so is that researchers do not have to make any assumptions regarding the minimal clinically important difference or the expected variability of the data.

The Variability of the Data

For quantitative data, researchers also need to determine the expected variability of the alternative treatments: the more variability expected in the specified outcome, the more difficult it will be to differentiate between treatments and the larger the required sample size (see example below). If this variability is underestimated at the time of planning, the sample size computed will be too small and the study will be underpowered to the one desired. For comparing proportions, the calculation of sample size makes use of the expected proportion with the specified outcome in each group. For survival data, the calculation of sample size is based on the survival proportions in each treatment group at a specified time and on the total number of events in the group in which the fewer events occur. Therefore, for the latter two types of data, variability does not appear in the computation of sample size.

Presume an investigator wants to compare the postoperative Harris hip score [ 12 ] at 3 months in a group of patients undergoing minimally invasive THA with a control group of patients undergoing standard THA in a randomized controlled trial. The investigator must (1) establish a statistical significance level, eg, α = 5%, (2) select a power, eg, 1 − β = 90%, and (3) establish a targeted difference in the mean scores, eg, 10, and assume a standard deviation of the scores, eg, 20 in both groups (which they can obtain from the literature or their previous patients). In this case, the sample size should be 85 patients per group (Appendix 1). If fewer patients are included in the trial, the probability of detecting the targeted difference when it exists will decrease; for sample sizes of 64 and 32 per group, for instance, the power decreases to 80% and 50%, respectively (Fig.  1 ). If the investigator assumed the standard deviation of the scores in each group to be 30 instead of 20, a sample size of 190 per group would be necessary to obtain a power of 90% with a significance level α = 5% and targeted difference in the mean scores of 10. If the significance level was chosen at α = 1% instead of α = 5%, to yield the same power of 90% with a targeted difference in scores of 10 and standard deviation of 20, the sample size would increase from 85 patients per group to 120 patients per group. In relatively simple cases, statistical tables [ 19 ] and dedicated software available from the internet may be used to determine sample size. In most orthopaedic clinical trials cases, sample size calculation is rather simple as above, but it will become more complex in other cases. The type of end points, the number of groups, the statistical tests used, whether the observations are paired, and other factors influence the complexity of the calculation, and in these cases, expert statistical advice is recommended.

Sample Size, Estimation, and Regression

Sample size was presented above in the context of hypothesis testing. However, it is also of interest in other areas of biostatistics, such as estimation or regression. When planning an experiment, researchers should ensure the precision of the anticipated estimation will be adequate. The precision of an estimation corresponds to the width of the confidence interval: the larger the tested sample size is, the better the precision. For instance, Handl et al. [ 11 ], in a biomechanical study of 21 fresh-frozen cadavers, reported a mean ultimate load failure of four-strand hamstring tendon constructs of 4546 N under loading with a standard deviation of 1500 N. Based on these values, if we were to design an experiment to assess the ultimate load failure of a particular construct, the precision around the mean at the 95% confidence level would be expected to be 3725 N for five specimens, 2146 N for 10 specimens, 1238 N for 25 specimens, 853 N for 50 specimens, and 595 N for 100 specimens tested (Appendix 2); if we consider the estimated mean will be equal to 4546 N, the one obtained in the previous experiment, we could obtain the corresponding 95% confidence intervals (Fig.  2 ). Because we always deal with limited samples, we never exactly know the true mean or standard deviation of the parameter distribution; otherwise, we would not perform the experiment. We only approximate these values, and the results obtained can vary from the planned experiment. Nonetheless, what we identify at the time of planning is that testing more than 50 specimens, for instance 100, will multiply the costs and time necessary to the experiment while providing only slight improvement in the precision.

An external file that holds a picture, illustration, etc.
Object name is 11999_2008_346_Fig2_HTML.jpg

The graph shows the predicted confidence interval for experiments with an increasing number of specimens tested based on the study by Handl et al. [ 11 ] of 21 fresh-frozen cadavers with a mean ultimate load failure of four-strand hamstring tendon constructs of 4546 N and standard deviation of 1500 N.

Similarly, sample size issues should be considered when performing regression analyses, namely, when trying to assess the effect of a particular covariate, or set of covariates, on an outcome. The effective power to detect the significance of a covariate in predicting this outcome depends on the outcome modeled [ 14 , 30 ]. For instance, when using a Cox regression model, the power of the test to detect the significance of a particular covariate does not depend on the size of the sample per se but on the number of specific critical events. In a cohort study of patients treated for soft tissue sarcoma with various treatments, such as surgery, radiotherapy, chemotherapy, etc, the power to detect the effect of chemotherapy on survival will depend on the number of patients who die, not on the total number of patients in the cohort. Therefore, when planning such studies, researchers should be familiar with these issues and decide, for example, to model a composite outcome, such as event-free survival that includes any of the following events: death from disease, death from other causes, recurrence, metastases, etc, to increase the power of the test.

The reasons to plan a trial with an adequate sample size likely to give enough power to detect a meaningful difference are essentially ethical. Small trials are considered unethical by most, but not all, researchers because they expose participants to the burdens and risks of human research with a limited chance to provide any useful answers [ 2 , 10 , 28 ]. Underpowered trials also ineffectively consume resources (human, material) and add to the cost of healthcare to society. Although there are particular cases when trials conducted on a small sample are justified, such as early-phase trials with the aim of guiding the conduct of subsequent research (or formulating hypotheses) or, more rarely, for rare diseases with the aim of prospectively conducting meta-analyses, they generally should be avoided [ 10 ]. It is also unethical to conduct trials with too large a sample size because, in addition to the waste of time and resources, they expose participants in one group to receive inadequate treatment after appropriate conclusions should have been reached. Interim analyses and adaptive trials have been developed in this context to shorten the time to decision and overcome these concerns [ 4 , 16 ].

We raise two important points. First, we explained, for practical and ethical reasons, experiments are conducted on a sample of limited size with the aim to generalize the results to the population of interest and increasing the size of the sample is a way to combat uncertainty. When doing this, we implicitly consider the patients or specimens in the sample are randomly selected from the population of interest, although this is almost never the case; even if it were the case, the population of interest would be limited in space and time. For instance, Marx et al. [ 20 ], in a survey conducted in late 1998 and early 1999, assessed the practices for anterior cruciate ligament reconstruction on a randomly selected sample of 725 members of the American Academy of Orthopaedic Surgeons; however, because only ½ the surgeons responded to the survey, their sample probably is not representative of all members of the society, who in turn are not representative of all orthopaedic surgeons in the United States, who again are not representative of all surgeons in the world because of the numerous differences among patients, doctors, and healthcare systems across countries. Similar surveys conducted in other countries have provided different results [ 17 , 22 ]. Moreover, if the same survey was conducted today, the results would possibly differ. Therefore, another source for variation among studies, apart from sampling variability, is that samples may not be representative of the same population. Therefore, when planning experiments, researchers must take care to make their sample representative of the population they want to infer to and readers, when interpreting the results of a study, should always assess first how representative the sample presented is regarding their own patients. The process implemented to select the sample, the settings of the experiment, and the general characteristics and influencing factors of the patients must be described precisely to assess representativeness and possible selection biases [ 7 ].

Second, we have discussed only sample size for interpreting nonsignificant p values, but it also may be of interest when interpreting p values that are significant. Significant results issued from larger studies usually are given more credit than those from smaller studies because of the risk of reporting exaggerating treatment effects with studies with smaller samples or of lower quality [ 23 , 27 ], and small trials are believed to be more biased than others. However, there is no statistical reason a significant result in a trial including 2000 patients should be given more belief than a trial including 20 patients, given the significance level chosen is the same in both trials. Small but well-conducted trials may yield a reliable estimation of treatment effect. Kjaergard et al. [ 18 ], in a study of 14 meta-analyses involving 190 randomized trials, reported small trials (fewer than 1000 patients) reported exaggerated treatment effects when compared with large trials. However, when considering only small trials with adequate randomization, allocation concealment (allocation concealment is the process that keeps clinicians and participants unaware of upcoming assignments. Without it, even properly developed random allocation sequences can be subverted), and blinding, this difference became negligible. Nonetheless, the advantages of a large sample size to interpret significant results are it allows a more precise estimate of the treatment effect and it usually is easier to assess the representativeness of the sample and to generalize the results.

Sample size is important for planning and interpreting medical research and surgeons should become familiar with the basic elements required to assess sample size and the influence of sample size on the conclusions. Controlling for the size of the sample allows the researcher to walk a thin line that separates the uncertainty surrounding studies with too small a sample size from studies that have failed practical or ethical considerations because of too large a sample size.

Acknowledgments

We thank the editor whose thorough readings of, and accurate comments on drafts of the manuscript have helped clarify the manuscript.

The sample size (n) per group for comparing two means with a two-sided two-sample t test is

equation M1

where z 1−α/2 and z 1−β are standard normal deviates for the probability of 1 − α/2 and 1 − β, respectively, and d t  = (μ 0  − μ 1 )/σ is the targeted standardized difference between the two means.

The following values correspond to the example:

  • α = 0.05 (statistical significance level)
  • β = 0.10 (power of 90%)
  • |μ 0  − μ 1 | = 10 (difference in the mean score between the two groups)
  • σ = 20 (standard deviation of the score in each group)
  • z 1−α/2  = 1.96
  • z 1−β  = 1.28

equation M2

Two-sided tests which do not assume the direction of the difference (ie, that the mean value in one group would always be greater than that in the other) are generally preferred. The null hypothesis makes the assumption that there is no difference between the treatments compared, and a difference on one side or the other therefore is expected.

Computation of Confidence Interval

To determine the estimation of a parameter, or alternatively the confidence interval, we use the distribution of the parameter estimate in repeated samples of the same size. For instance, consider a parameter with observed mean, m, and standard deviation, sd, in a given sample. If we assume that the distribution of the parameter in the sample is close to a normal distribution, the means, x n , of several repeated samples of the same size have true mean, μ, the population mean, and estimated standard deviation,

equation M3

also known as standard error of the mean, and

equation M4

follows a t distribution. For a large sample, the t distribution becomes close to the normal distribution; however, for a smaller sample size the difference is not negligible and the t distribution is preferred. The precision of the estimation is

equation M5

For example, Handl et al. [ 11 ] in a biomechanical study of 21 fresh-frozen cadavers reported a mean ultimate load failure of 4-strand hamstring tendon constructs of 4546 N under dynamic loading with standard deviation of 1500 N. If we were to plan an experiment, the anticipated precision of the estimation at the 95% level would be

equation M7

for five specimens,

equation M8

The values 2.78, 2.26, 2.06, 2.01, and 1.98 correspond to the t distribution deviates for the probability of 1 − α/2, with 4, 9, 24, 49, and 99 (n − 1) degrees of freedom; the well known corresponding standard normal deviate is 1.96. Given an estimated mean of 4546 N, the corresponding 95% confidence intervals are 2683 N to 6408 N for five specimens, 3473 N to 5619 N for 10 specimens, 3927 N to 5165 N for 25 specimens, 4120 N to 4972 N for 50 specimens, and 4248 N to 4844 N for 100 specimens (Fig.  2 ).

Similarly, for a proportion p in a given sample with sufficient sample size to assume a nearly normal distribution, the confidence interval extends either side of the proportion p by

equation M12

For a small sample size, exact confidence interval for proportions should be used.

Each author certifies that he or she has no commercial associations (eg, consultancies, stock ownership, equity interest, patent/licensing arrangements, etc) that might pose a conflict of interest in connection with the submitted article.

Stack Exchange Network

Stack Exchange network consists of 183 Q&A communities including Stack Overflow , the largest, most trusted online community for developers to learn, share their knowledge, and build their careers.

Q&A for work

Connect and share knowledge within a single location that is structured and easy to search.

Effect of sample size on practical and statistical significance

I am very confused about the effect of sample size on statistical and practical significance levels. I have two questions about them: 1. Can somebody explain the p value variation with sample size intuitively and mathematically. 2. Lets say I increase my practical significance from 2% to 3%. How would that effect the sample size requirement for the same power.

I have browsed internet for them but have gotten more confused. Can somebody please explain these in intuitively and mathematically ?

  • hypothesis-testing
  • statistical-significance
  • experiment-design
  • mathematical-statistics

Michael R. Chernick's user avatar

  • 1 $\begingroup$ Practical significance depends on what an important difference is for your problem. It is not something that you should change. Statistical significance is a difference that you can choose. Often practical significance can be obtained with a smaller sample size. $\endgroup$ –  Michael R. Chernick Commented Apr 1, 2017 at 19:11

In your question you write that you are confused, so I will try to keep this answer as close to a general understanding as possible. My definitions are made to give you an intuition of what is going. From a strict statistical perspective they are actually very inaccurate and maybe you will see in a few months why. But let's start now:

Practical significance is a matter of what significance means in a general meaning. For example, you could conduct an experiment with 100.000 participants. Half of them gets a chewing gum under their shoes and you compare their walking speed with the control group, who don't have chewing gum under their shoes. You can observe that the average walking speed for the chewing gum group is 5.00000 km/h and the control group has an average of 5.00001 km/h (which might be statistically significant). Ask your personal reasoning: would this be a sufficient result to forbid chewing gums? That's the question practical significance answers you. Mostly this may be measured using so called effect sizes .

Statistical significance is a calculation you do to check wether your results are really due to chance or a systematic effect (this is not exactly accurate but in a very very loose definition, this is what it's all about). For example you may ask two women and two men for their IQ. The average for men is 95 and for women 105 (fictional example). How strongly would you claim that there is a difference. Maybe you just got two women with above average intelligence?! Now consider you sample 100'000 women and men and get the same result. Which survey would you trust more? Probably the latter one. This is what statistical significance is about. Now imagine you have to predict the next elections. Party A and Party B both have 45% in the polls. You will probably not be able to say that very easily who will win. On the other hand consider party A has 70% in the polls and party B 10%. It is fairly easy to see that Party A will probably win, isn't it? This difference between the two things you compare is the effect . The more they differ the easier to show that they are different. Now let us consider Party A has 55% and Party B 40%. Now I ask you three questions. 1) what is your guess who will win? 2) can you tell me who will most likely win? 3) can you tell me with absolute certainty who will win? You may answer 1) with A might win and 2) with A will probably win. At least their odds are better at the moment than for Party B. Question 3) you would not be able to answer at all. Do you see that with the increasing level of required certainty or confidence it become harder to give a sound answer / result?

Now imagine a pharmaceutical company which has been in the press because their product has been suspected to cause cancer for 1 in 20 patients. Three days later the PR department issues a statement "we conducted a study among 3 participants and haven't found any negative side effect!" Would you say that 3 people is enough to really find an effect? Or might it be that you actually have a low chance of finding an effect for 1 in 20 people if you only sample 3 persons? That is what power tells you: the probability of finding an effect if it really exists. In this example, you could certainly say: they just didn't try enough!

The summary: General significant depends on the effect but not sample size.

Statistical significance gets better ( p value smaller) with more people ( larger sample), a greater difference (effect) between the groups and a lower required level of certainty ( lower confidence level).

Power increases with larger sample size, larger effect and smaller confidence.

The general discussion depends on understanding null hypothesis, alternate hypothesis and the decision which hypothesis is better supported by the data. You must know what type I and type II errors are. Then you will understand that significance is broadly about controlling type I error rates and power the opposite to type II errors. However, to explain all that is beyond the scope of this post.

For the mathematical calculation I find it difficult to show you what's going on since I do not know which distributions you know, and if you know about normalisation. Do you know about limits? That would certainly help your understanding of the mathematical part of all questions about what influences significance.

Carol Eisen's user avatar

Your Answer

Sign up or log in, post as a guest.

Required, but never shown

By clicking “Post Your Answer”, you agree to our terms of service and acknowledge you have read our privacy policy .

Not the answer you're looking for? Browse other questions tagged hypothesis-testing statistical-significance experiment-design mathematical-statistics or ask your own question .

  • Featured on Meta
  • Announcing a change to the data-dump process
  • Bringing clarity to status tag usage on meta sites

Hot Network Questions

  • What's the benefit or drawback of being Small?
  • What other crewed spacecraft returned uncrewed before Starliner Boe-CFT?
  • Replacing jockey wheels on Shimano Deore rear derailleur
  • What would be a good weapon to use with size changing spell
  • Why would op-amp shoot up to positive rail during power on?
  • Is it a good idea to perform I2C Communication in the ISR?
  • Could a lawyer agree not to take any further cases against a company?
  • What is the nature of the relationship between language and thought?
  • What does "Two rolls" quote really mean?
  • Is it helpful to use a thicker gage wire for part of a long circuit run that could have higher loads?
  • Why does this theta function value yield such a good Riemann sum approximation?
  • How to run only selected lines of a shell script?
  • Is there a problem known to have no fastest algorithm, up to polynomials?
  • Text wrapping in longtable not working
  • What is this movie aircraft?
  • How do I learn more about rocketry?
  • A seven letter *
  • Why would autopilot be prohibited below 1000 AGL?
  • What was the first "Star Trek" style teleporter in SF?
  • Can I use Cat 6A to create a USB B 3.0 Superspeed?
  • do-release-upgrade from 22.04 LTS to 24.04 LTS still no update available
  • Book about a wormhole found inside the Moon
  • How to change upward facing track lights 26 feet above living room?
  • How rich is the richest person in a society satisfying the Pareto principle?

how does sample size influence hypothesis testing results

IMAGES

  1. How to determine correct sample size for hypothesis testing?

    how does sample size influence hypothesis testing results

  2. PPT

    how does sample size influence hypothesis testing results

  3. Everything You Need To Know about Hypothesis Testing

    how does sample size influence hypothesis testing results

  4. PPT

    how does sample size influence hypothesis testing results

  5. Hypothesis Testing Concept Map

    how does sample size influence hypothesis testing results

  6. PPT

    how does sample size influence hypothesis testing results

VIDEO

  1. How to calculate/determine the Sample size for difference in proportion/percentage between 2 groups?

  2. How to determine correct sample size for hypothesis testing?

  3. Chapter 8: Introduction to Hypothesis Testing (Section 8-4, 8-5, and 8-6)

  4. Hypothesis Testing 🔥 Explained in 60 Seconds

  5. Hypothesis testing in Large Samples-V: Sample and the Population Standard Deviations

  6. Sample Size Calculation || Case Control Study Design || Alpha, Beta, Test Power

COMMENTS

  1. Sample Size and its Importance in Research

    In hypothesis testing studies, this is mathematically calculated, conventionally, as the sample size necessary to be 80% certain of identifying a statistically significant outcome should the hypothesis be true for the population, with P for statistical significance set at 0.05. Some investigators power their studies for 90% instead of 80%, and ...

  2. Sample size, power and effect size revisited: simplified and practical

    Sample size, power and effect size revisited: simplified ... - NCBI

  3. How sample size influences research outcomes

    An appropriate sample renders the research more efficient: Data generated are reliable, resource investment is as limited as possible, while conforming to ethical principles. The use of sample size calculation directly influences research findings. Very small samples undermine the internal and external validity of a study.

  4. The Relationship between Significance, Power, Sample Size & Effect Size

    The sample size or the number of participants in your study has an enormous influence on whether or not your results are significant. The larger the actual difference between the groups (ie. student test scores) the smaller of a sample we'll need to find a significant difference (ie. p ≤ 0.05).

  5. 11.8: Effect Size, Sample Size and Power

    The answer, shown in Figure 11.5, is that almost the entirety of the sampling distribution has now moved into the critical region. Therefore, if θ=0.7 the probability of us correctly rejecting the null hypothesis (i.e., the power of the test) is much larger than if θ=0.55. In short, while θ=.55 and θ=.70 are both part of the alternative ...

  6. How to Calculate Sample Size Needed for Power

    Statistical power and sample size analysis provides both numeric and graphical results, as shown below. The text output indicates that we need 15 samples per group (total of 30) to have a 90% chance of detecting a difference of 5 units. The dot on the Power Curve corresponds to the information in the text output.

  7. Sample Size and Hypothesis Testing

    Hypothesis testing determines if there is sufficient evidence to support a claim (the statistical hypothesis) about a population parameter based on a sample of data. Right-sizing experiments involve trade-offs involving the probabilities of different kinds of false claims, precision of estimates, and operational and ethical constraints on ...

  8. 25.3

    Answer. Setting α, the probability of committing a Type I error, to 0.05, implies that we should reject the null hypothesis when the test statistic Z ≥ 1.645, or equivalently, when the observed sample mean is 103.29 or greater: because: x ¯ = μ + z (σ n) = 100 + 1.645 (16 64) = 103.29. Therefore, the power function \K (\mu)\), when μ ...

  9. PDF Hypothesis Testing: Significance, Effect Size, or and Confidence

    ut hypotheses, but also to help describe the nature of the effects being tested. Sig. ificance, effect size, and confidence intervals are three levels of information. Table 7.2 summarizes the information provided by each level, and ad. s a fourth le. el that is introduced in the next lea.

  10. Understanding Significance Levels in Statistics

    It decreases the sample size required in order to detect the difference as significant; and/or increases the power of the study to detect this difference as statistically significant. I like to think in terms of all three: power, alpha, and sample size related to your effect size. The free software G*Power does a good job of determining these.

  11. Issues in Estimating Sample Size for Hypothesis Testing

    Suppose we want to test the following hypotheses at aα=0.05: H 0: μ = 90 versus H 1: μ ≠ 90. To test the hypotheses, suppose we select a sample of size n=100. For this example, assume that the standard deviation of the outcome is σ=20. We compute the sample mean and then must decide whether the sample mean provides evidence to support the ...

  12. 8.3: Sampling distribution and hypothesis testing

    Results from Shapiro-Wilks test: W = 0.97426, p-value = 0.04721. And finally, modify the code to draw one million samples, we get: Figure \(\PageIndex{1}\): Means of one million replicate samples drawn at random from chi-square distribution, df = 1. Normality test will fail to run, sample size of 5000 limit.

  13. Large sample size, significance level, and the effect size: Solutions

    For example, with a sample size of 50, if we investigate a relationship with an effect size of 0.80, the test power (1 - β) to reject the null hypothesis will be approximately 0.85, whereas with the same sample size, if we investigate a relationship with an effect size of 0.20, the test power (1 - β) to reject the null hypothesis will be ...

  14. Hypothesis Testing, P Values, Confidence Intervals, and Significance

    Hypothesis testing allows us to determine the size of the effect. An example of findings reported with p values are below: Statement: Drug 23 reduced patients' symptoms compared to Drug 22. Patients who received Drug 23 (n=100) were 2.1 times less likely than patients who received Drug 22 (n = 100) to experience symptoms of Disease A, p<0.05. Or

  15. The Importance and Effect of Sample Size

    A greater power requires a larger sample size. Effect size - This is the estimated difference between the groups that we observe in our sample. To detect a difference with a specified power, a smaller effect size will require a larger sample size. When conducting research about your customers, patients or products it's usually impossible, or ...

  16. Why sample size and effect size increase the power of a statistical test

    How we test this hypothesis is that we calculate the test statistics and p value then compare with α. z = (168-165) ÷ (7.2/( square root of sample size )) = (168-165)× square root of sample ...

  17. Optimal Significance Level and Sample Size in Hypothesis Testing. 1

    The significance level has been classically considered to be 5% (following the initial suggestion provided by Ronald Fisher) and while other typical values are sometimes employed (e.g. 10%, 1%, 0. ...

  18. Statistical Significance, Sample Size and Expected Effects

    Power. The results of your experiment are validated and can be accepted only if the results for the given experiment pass a significance test.The sample size is adjusted using statistical power.. For example, if an experimenter takes a survey of a group of 100 people and decides the presidential votes based on this data, the results are likely to be highly erroneous because the population size ...

  19. Type I & II Errors and Sample Size Calculation in Hypothesis Testing

    Photo by Scott Graham on Unsplash. In the world of statistics and data analysis, hypothesis testing is a fundamental concept that plays a vital role in making informed decisions. In this blog, we will delve deeper into hypothesis testing, specifically focusing on how to reduce type I and type II errors.We will discuss the factors that influence these errors, such as significance level, sample ...

  20. Step-by-step guide to hypothesis testing in statistics

    Misunderstanding 4: Hypothesis Testing Guarantees Accurate Results. Clarification: Hypothesis testing helps you make decisions based on data, but it doesn't guarantee your results are correct. The quality of your data and the right choice of test affect how reliable your results are.

  21. Interpreting Results from Statistical Hypothesis Testing: Understanding

    There are two types of power analyses, the a priori method and the post hoc procedure. The a priori method determines the power and ES before the study and determines the sample size required for the null hypothesis significance test in advance. Because the power is fixed at 0.8, the sample size can be obtained if the ES is known in advance.

  22. Level of Significance & Hypothesis Testing

    In hypothesis testing, the level of significance is a measure of how confident you can be about rejecting the null hypothesis. This blog post will explore what hypothesis testing is and why understanding significance levels are important for your data science projects. In addition, you will also get to test your knowledge of level of significance towards the end of the blog with the help of quiz.

  23. hypothesis testing

    Placidia. 14.5k 6 42 73. 1. In a two sample situation, increasing the sample size of one group to infinity does not send the power of the test to 1. The power will be limited by the sample size of the smaller group (or, to be precise, a combination of the variances within the groups and the sample sizes, if you think about a t t -test).

  24. Statistics in Brief: The Importance of Sample Size in the Planning and

    The graphs show the distribution of the test statistic (z-test) for the null hypothesis (plain line) and the alternative hypothesis (dotted line) for a sample size of (A) 32 patients per group, (B) 64 patients per group, and (C) 85 patients per group.For a difference in mean of 10, a standard deviation of 20, and a significance level α of 5%, the power (shaded area) increases from (A) 50%, to ...

  25. hypothesis testing

    I am very confused about the effect of sample size on statistical and practical significance levels. I have two questions about them: 1. Can somebody explain the p value variation with sample size intuitively and mathematically. 2. Lets say I increase my practical significance from 2% to 3%.